Medical researcher

This topic has expert replies
Master | Next Rank: 500 Posts
Posts: 176
Joined: Thu Sep 22, 2011 5:32 am
Thanked: 5 times

Medical researcher

by vishal.pathak » Mon Mar 26, 2012 12:17 am
Medical researcher: As expected, records covering the last four years often major hospitals indicate that babies born prematurely were more likely to have low birth weights and to suffer from health problems than were babies not born prematurely. These records also indicate that mothers who had received adequate prenatal care were less likely to have low birth weight babies than were mothers who had received inadequate prenatal care. Adequate prenatal care, therefore, Significantly decreases the risk of low birth weight babies.
Which one of the following, if true, most weakens the medical researcher's argument'?
(A) The hospital records indicate that many babies that are born with normal birth weights are born to mothers who had inadequate prenatal care.
(B) Mothers giving birth prematurely are routinely classified by hospitals as having received inadequate prenatal care when the record of that care is not available.
(C) The hospital records indicate that low birth weight babies were routinely classified as having been born prematurely.
(D) Some babies not born prematurely whose mothers received adequate prenatal care, have low birth weights.
(E) Women who receive adequate prenatal care are less likely to give birth prematurely than are women who do not receive adequate prenatal care.
[spoiler]OA B. This is a typical cause-effect question. Here cause is adequate prenatal care and effect is reduced risk of low birth weight babies
Choice A shows that there is an alternate way to get the effect. This question is from powerscore and the explanation given there is that the stimulus does not say that adequate prenatal care is the only way to get healthy babies. I believe that had the stimulus stated that adequate prenatal care is the only way to get healthy babies then this choice would have completely destroyed the argument but even in its present form it makes sufficient dent to the argument.

Thoughts?? [/spoiler]

User avatar
GMAT Instructor
Posts: 88
Joined: Tue Jan 17, 2012 5:01 pm
Thanked: 54 times
Followed by:37 members

by chris@magoosh » Mon Mar 26, 2012 9:09 am
Let's say (A) does make a sufficient dent to the question - it still doesn't answer the question:

"Which one of the following, if true, most weakens the medical researcher's argument."

(B) on the other hand completely weakens (vs. dents) the conclusion. Mothers who give birth to premature babies, but who do not have record have that care, are automatically grouped in the "inadequate care" category, thus conflating low birth weight of premature babies with all babies receiving inadequate care.

Returning to (A), it simply states that many normal birth weight babies received inadequate care. That doesn't mean that the inadequate care group doesn't have a lower risk than the adequate care group. The latter group could have had a greater percentage of babies born at a normal birth weight.

So (B) is the clear cut answer.

Hope that helps :).

Master | Next Rank: 500 Posts
Posts: 341
Joined: Sun Mar 25, 2012 6:59 pm
Thanked: 17 times
Followed by:4 members
GMAT Score:720

by ice_rush » Mon Mar 26, 2012 12:06 pm
Hi Chris,
I was all for picking (B) until I read the part....when the record of that care is not available. We don't know the number of patients for whom the record of care was not available....It could be only 2 patients or 1000, who knows? A bigger number would definitely have a significant impact on the argument, but a small number -- I am not too sure. Please advise.



Thanks!

Master | Next Rank: 500 Posts
Posts: 176
Joined: Thu Sep 22, 2011 5:32 am
Thanked: 5 times

by vishal.pathak » Mon Mar 26, 2012 2:52 pm
ice_rush wrote:Hi Chris,
I was all for picking (B) until I read the part....when the record of that care is not available. We don't know the number of patients for whom the record of care was not available....It could be only 2 patients or 1000, who knows? A bigger number would definitely have a significant impact on the argument, but a small number -- I am not too sure. Please advise.



Thanks!
But the option also says "routinely", so this means that they classify any baby born prematurely as the one who has not got adequate care